Write a recursive formula for the following arithmetic sequence.1, 12, -25, -38, ...a 1=an= for n ≧2

Write A Recursive Formula For The Following Arithmetic Sequence.1, 12, -25, -38, ...a 1=an= For N 2

Answers

Answer 1

First, let's see the progression of the sequence of numbers

As you can see from the second term the sequence continues with a constant change of -13 which means d=-13.

The answers will be the expressions inside the red square box in each case.

Write A Recursive Formula For The Following Arithmetic Sequence.1, 12, -25, -38, ...a 1=an= For N 2
Write A Recursive Formula For The Following Arithmetic Sequence.1, 12, -25, -38, ...a 1=an= For N 2

Related Questions

PLEASE HELP!!!!!

Answer two questions about Equations AAA and BBB:

A. 4x+2=6-x
B. 5x+2=6

1) How can we get Equation BBB from Equation AAA?

Choose 1 answer:

(Choice A)
Add/subtract a quantity to/from only one side

(Choice B)
Add/subtract the same quantity to/from both sides

(Choice C)
Multiply/divide only one side by a non-zero constant

(Choice D)
Multiply/divide both sides by the same non-zero constant

Based on the previous answer, are the equations equivalent? In other words, do they have the same solution?

Choose 1 answer:

(Choice A)
Yes

(Choice B)
No

Answers

We can get equation B from equation A by Adding the same quantity to both sides. The equations are equivalent and have the same solution.

The equations are:

A : 4x+2 = 6-x

B : 5x+2 = 6

(1) We can get equation B from equation A by adding x to both sides of the equation.

We can verify this.

Add x to both sides of the equation A,

⇒ 4x+2 +x = 6-x+x

⇒ 5x+2 = 6

This is equation B.

So Add the same quantity to both sides of equation A to get equation B.

(2) We just added a same quantity on both sides of equation A. This will not change the equations. So equation A and equation B are equivalent. Also they have same solution.

Equation A: 4x+2=6-x

⇒ 4x+x = 6-2

⇒ 5x = 4

⇒ x = 4/5

Equation B: 5x+2 = 6

⇒ 5x = 6-2

⇒ 5x = 4

⇒ x = 4/5

So both have same solution.

Learn more about equivalent equation at https://brainly.com/question/2972832

#SPJ1

make a 2 column proofplease make it simple like JK is parallel to NM(given)

Answers

Explanation:

Since L is the midpoint of JM, then JL = LM.

Therefore,

Statement: JL = LM

Reason: L is the midpoint of JM

The lines JK and NM are parrallel; therefore, by the alternate interior angles theorem,

[tex]\angle LJK=\angle LMN[/tex]

Furthermore, since ∠JLK and ∠MLN are vertical angles,

[tex]∠JLK=∠MLN[/tex]

Now since ∠JLK = ∠MLN, ∠LJK = ∠LMN, and JL = LM, then by ASA postulate

[tex]\boxed{△JKL=△MNL.}[/tex]

Hence, our proof is complete!

which transformations had to occur for the blue triangle to become the purple triangle? [note: all rotations are about origin]

Answers

Let:

A = (-4,3)

B = (-1,3)

C = (-1,1)

after a 90 clockwise rotation:

A = (-4,3)--------->(y,-x)------->(3,4)

B = (-1,3)----------->(y,-x)------>(3,1)

C = (-1,1)------------>(y,-x)------>(1,1)

After a translation 3 units down and 2 units left:

(3,4)------>(x-2,y-3)------->(1,1)

(3,1)------>(x-2,y-3)------->(1,-2)

(1,1)------>(x-2,y-3)------->(-1,-2)

---------------------------

Therefore, the answer is D

Donny the Dot Dude is saving for a llama. He puts$3,000 in a savings account that earns 12% simpleannual interest. How many years it will it take forhim to have the $4,440 he needs ifhe makes no additional depositsor withdrawals?A) 4 yearsSOB) 5 yearsC) 6 yearseboD) 2 yearsх

Answers

The simple interest formula is:

A = P(1 + rt)

where A is the final amount, P is the princiapal, r is the annual interest rate (as a decimal), and t is time in years.

Substituting with A = $4,440, P = $3,000, and r = 0.12 (= 12/100), we get:

[tex]\begin{gathered} 4440=3000\cdot(1+0.12\cdot t) \\ \frac{4440}{3000}=1+0.12\cdot t \\ 1.48-1=0.12\cdot t \\ \frac{0.48}{0.12}=t \\ 4\text{ years = t} \end{gathered}[/tex]

The school record for the greatest number of jumping jacks in a row is 84 in 4 minutes. If the record for jumping jacks made is a constant ratio, how many jumping jacks did the record holder make in 1 minute?

Answers

The amount of jumping packs in 1 minute is 21

How to determine the amount of jumping packs in 1 minute?

From the question, the given parameters are:

Number of jumping packs in a row = 84 packs

Number of minutes = 4 minutes

The amount of jumping packs in 1 minute is the quotient of the number of jumping packs in a row and the number of minutes

This is represented as

Jumping packs in 1 minute = Number of jumping packs in a row /Number of minutes

Substitute the known values in the above equation

Jumping packs in 1 minute = 84/4

Evaluate

Jumping packs in 1 minute = 21

Hence, the jumping packs in 1 minute is 21

Read more about unit rates at

https://brainly.com/question/4895463

#SPJ1

The record holder makes 21 jumping jacks in 1 minute which is the ratio of the number of jumping jacks performed in a row to the total number of minutes.

What is the ratio?

A ratio is a relationship between two amounts that is represented by the division of one by the other.

The ratio of the total number of jumping jacks performed in a row to the total number of minutes determines how many jumping jacks should be performed in a minute.

No. of jumping packs in a row = 84 packs

No. of minutes = 4 minutes

So jumping packs in 1 minute = 84/4

Apply the division operation to get

Jumping packs in 1 minute = 21

Therefore, 21 jumping jacks should be performed in 1 minute.

Learn more about the Ratios here:

brainly.com/question/1504221

#SPJ1

someone help please

Answers

Answer:

Angle 3 = 95° (vertically opposite angles)

Angle 2 and 4 = 180 - 95

= 85° (adjacent angles on straight line)

Angle 5 = 180 - 144

= 36° (adjacent angles on straight line)

Angle 6 = 180 - angle 3 - angle 5

= 180 - 95 - 36

= 49° (sum of angles in triangle=180)

Angle 1 = 180 - 90 - angle 6

= 180 - 90 - 49

= 41° (sum of angles in triangle=180)

Angle 7 = 180 - 38 - angle 5

= 180 - 38 - 36

= 106° (sum of angles in triangle=180)

Find the present value given the following:Amount needed: $9,350Time in years: 3Interest: 5%Compounded: semiannually

Answers

Solution:

Amount needed (P): $9,350

Time in years (n): 3

Interest (r): 5%

Compounded: semiannually

To find the Amount (A).

we have the formula A, we get,

[tex]A=P(1+\frac{\frac{r}{2}}{100})^{2n}[/tex][tex]=9350(1+\frac{2.5}{100})^{2(3)}[/tex][tex]=9350(\frac{102.5}{100})^6[/tex][tex]=9350(1.025)^6[/tex][tex]=9350(1.15969)[/tex][tex]A=10,843.13[/tex]

The present value is $10,843.133

ILL GIVE BRAINLIEST!!!!

Answers

Answer:

1 = yes

2 = no

3 = no

4 = yes

5 = yes

6 = no

7 = no

8 = no

7. Solve for value of tan A 8. Solve for Cos Q

Answers

7. Tan A = opposite/adjecent

Therefore Tan A = 24/7

• Option number (2) is correct .

8. Cos Q = adjacent/hypotaneus

where hypotaneus side : r^2 = x^2+y^2

r^2 = 12^2 +5^2= 169

r =√169 = 13

Now Cos Q = 12/13

• Option 4 is correct .

Middle School Debate Club 30% members are in 6th grade. If there are 12 6th graders in The Debate Club, how many total members are there ?

Answers

We know that 30% of the members are in 6th grade, and add up to 12 members.

Lets X be the total members.

Then 0.3*X are in 6th grade, and this is equivalent to 12 members.

NOTE: 0.3 is the decimal form of 30%.

We can write:

[tex]\begin{gathered} 0.3\cdot X=12 \\ X=\frac{12}{0.3} \\ X=40 \end{gathered}[/tex]

Answer: there are 40 members in the Debate Club.

How long ago, to the nearest year, was the artifact made?

Answers

Let's use the following formula:

[tex]A=A_0(0.5)^{\frac{t}{h}}[/tex]

where:

Ao= Initial amount

t = time

h = half-life

[tex]\begin{gathered} A=0.2A_0 \\ so\colon \\ 0.2A_0=A_0(0.5)^{\frac{t}{5730}} \end{gathered}[/tex]

solve for t:

[tex]\begin{gathered} 0.2=0.5^{\frac{t}{5730}} \\ \ln (0.2)=\frac{t}{5730}\ln (0.5) \\ t=5730\cdot\frac{\ln (0.2)}{\ln (0.5)} \\ t\approx13305 \end{gathered}[/tex]

Help me asp please!!

Answers

It is a function because it passes the vertical line test. The vertical line test says that if a vertical line is drawn on the graph anywhere, only one point of the graph will intersect with it.

I got -52.7 degrees but I want to make sure. thank you

Answers

The given vector is

[tex]m=\langle16,-21\rangle[/tex]

The formula to find the direction angle is

[tex]\theta=\tan ^{-1}|\frac{y}{x}|[/tex]

Replacing each coordinate, we have

[tex]\theta=\tan ^{-1}|\frac{-21}{16}|=52.7[/tex]Therefore, the direction angle is 52.7°, approximately.

12 is 58% of what number?

Also can you explain how to solve these problems?

Answers

Answer:

20.68976

Step-by-step explanation:

Convert the percentage into decimal :

58% = 0.58

0.58 × x = 12

Divide both sides by 0.58 :

x = 12÷0.58

x = 20.689655...

x = 20.68976

So the method to these types of question is to make the question into an equation by converting the percentage into a decimal, rearrange to make the unknown number the subject and solve .

Hope you understood and have a good day

How can you use the Power of a Quotient, Quotient of Powers, Zero Exponent Laws Identity Exponent and to evaluate numerical expressions with whole-number exponents?​

Answers

Exponents and powers are terms that occasionally get used interchangeably, which can be confusing.

Mathematics uses expressions called powers, where n is the exponent and x is the base. When a number or variable is multiplied repeatedly, it is referred to as a power. The exponent of power tells us how many times to multiply the base by itself.

You can interpret the term as "x to the power." The exponent (n) is written smaller and at the head of the line using superscript, whereas the base (x) is printed in full size (if you are typing it on a computer). For instance, it is written as x squared or x to the second power, which in reality means that the value of x is multiplied by an amount equal to the exponent's value.

If the base is a number: In this situation, all you have to do to discover the solution is multiply the base by itself as many times as the exponent's value.

If the base is a variable, you must first replace the variable with a value before continuing.

Learn more about powers and exponents at

https://brainly.com/question/15722035?referrer=searchResults

#SPJ1

What is a polygon with 10 sides called?dodecagonoctagontarragondecagon

Answers

A polygon with 10 sides is called a decagon.

x + 3 = -7 Solve the inequality

Answers

The given inequality is expressed as

[tex]\begin{gathered} x\text{ + 3 }\leq\text{ - 7} \\ x\text{ }\leq\text{ - 7 - 3} \\ x\text{ }\leq\text{ - 10} \end{gathered}[/tex]

A storage bin has the shape of a cylinder with a conical top. What is the volume of the storage bin if its radius is r=5.4 ft, the height of the cylindrical portion is h=7.7 ft, and the overall height is H=16.7 ft?

Answers

The Volume of a Compound Solid

The figure consists of a cylinder and a cone, both with the same radius of r=5.4 ft. The height of the cylinder is h=7.7 ft and the total height (of cone and cylinder) is H = 16.7 ft. This means the height of the cone is hc = 16.7 - 7.7 = 9 ft.

The volume of a cylinder of height h and radius r is:

[tex]V_{\text{cyl}}=\pi\cdot r^2\cdot h[/tex]

The volume of a cone of height hc and radius r is:

[tex]V_{\text{cone}}=\frac{\pi\cdot r^2\cdot h_c}{3}[/tex]

Calculate the volume of the cylinder:

[tex]\begin{gathered} V_{\text{cyl}}=\pi\cdot(5.4ft)^2\cdot7.7ft \\ V_{\text{cyl}}=705.388ft^3 \end{gathered}[/tex]

Calculate the volume of the cone:

[tex]V_{\text{cone}}=\frac{\pi\cdot(5.4ft)^2\cdot9}{3}=274.827ft^3[/tex]

Now we add both volumes:

V = 705.388 + 274.827 = 980.215 cubic feet

Rounding to the nearest tenth:

V = 980.2 cubic feet

What are the six trigonometric ratios, and how are some of them related to each other(which are reciprocals of which)?

Answers

Explanation:

Consider the following right triangle:

In this triangle

x = adjacent side to the angle theta.

y = opposite side to the angle theta.

h= hypotenuse.

Now, by definition, we have the following trigonometric ratios:

[tex]\cos(\theta)=\frac{adjacent\text{ side to the angle }\theta}{hypotenuse}\text{ =}\frac{x}{h}[/tex][tex]\sin(\theta)=\frac{opposite\text{ side to the angle }\theta}{hypotenuse}=\frac{y}{h}[/tex][tex]tan(\theta)=\frac{opposite\text{ side to the angle }\theta}{adjacent\text{ side to the angle }\theta}=\text{ }\frac{y}{x}=\frac{y\text{ /h}}{x\text{ /h}}\text{ =}\frac{\sin(\theta)}{\cos(\theta)}[/tex]

and according to the above trigonometric ratio, we get:

[tex]cotan(\theta)=\frac{\cos(\theta)}{\sin(\theta)}[/tex]

On the other hand, we get the following reciprocals:

[tex]csc(\theta)=\frac{1}{\sin(\theta)}[/tex]

and

[tex]sec(\theta)=\frac{1}{cos(\theta)}[/tex]

we can conclude that the correct answer is:

Answer:

The six trigonometric ratios:

[tex]\cos(\theta)=\frac{adjacent\text{ side to the angle }\theta}{hypotenuse}\text{ }[/tex]

[tex]\sin(\theta)=\frac{opposite\text{ side to the angle }\theta}{hypotenuse}[/tex]

[tex]tan(\theta)=\frac{opposite\text{ side to the angle }\theta}{adjacent\text{ side to the angle }\theta}=\text{ }\frac{\sin(\theta)}{\cos(\theta)}[/tex]

[tex]csc(\theta)=\frac{1}{\sin(\theta)}[/tex]

[tex]sec(\theta)=\frac{1}{cos(\theta)}[/tex]

[tex]cotan(\theta)=\frac{\cos(\theta)}{\sin(\theta)}[/tex]

1. What would the slope of a line that is parallel to the line in the graph be?
(4,3)
X

Answers

keeping in mind that parallel lines have exactly the same slope, let's check for the slope of the line above, since a parallel line will have the same slope anyway

[tex](\stackrel{x_1}{4}~,~\stackrel{y_1}{3})\qquad (\stackrel{x_2}{1}~,~\stackrel{y_2}{-1}) ~\hfill \stackrel{slope}{m}\implies \cfrac{\stackrel{rise} {\stackrel{y_2}{-1}-\stackrel{y1}{3}}}{\underset{run} {\underset{x_2}{1}-\underset{x_1}{4}}} \implies \cfrac{ -4 }{ -3 } \implies {\Large \begin{array}{llll} \cfrac{ 4 }{ 3 } \end{array}}[/tex]

Ana has two plants. From Monday to Tuesday, plant A grew 5 cm more than plant B. If the sum of the length of the two plants on Tuesday is 41 cm, how long is each plant on Tuesday?

Answers

Write a system of equations using A and B as the variables

The first equation is the sum of the lengths of the plants

[tex]A+B=41[/tex]

The second equation is the relation of growth that plant A grew 5cm more than plant B

[tex]A=B+5[/tex]

Insert the second equation into the first one to solve for B

[tex]\begin{gathered} A+B=41 \\ (B+5)+B=41 \end{gathered}[/tex]

Solve the equation for B

[tex]\begin{gathered} 2B+5=41 \\ 2B=41-5 \\ 2B=36 \\ B=\frac{36}{2} \\ B=18 \end{gathered}[/tex]

Use the value of B in the second equation to find the value of A

[tex]\begin{gathered} A=B+5 \\ A=18+5 \\ A=23 \end{gathered}[/tex]

Plant A is 23 cm long on Tuesday and plant B is 18 cm long on Tuesday.

v=LMH for L

thankssssss .

Answers

L= MH/ v Explanation : you move the v to divide it ( the reciprocal ) and then you’ll move the L to where the v was to replace it and that’s how you solve for L

A freshly brewed cup of coffee has temperature 95°C in a 20°C room. When its temperature is 67°C, it is cooling at a rate of 1°C per minute.

Let y = T − Ts, where T(t) is the temperature of the coffee in degrees Celsius at time t and Ts is the temperature of the surroundings in degrees Celsius. Find the values of A (in °C) and k for y(t) = Aekt.

Find: A and k

After how many minutes is the temperature of the coffee 67°C? (Round your answer to two decimal places.)
Answer:

Answers

The temperature of coffee will reach 67 °C

The rate of temperature change is most directly related to the difference between the body temperature and room temperature.

[tex]\frac{dT}{dt} =-k(T-T1)[/tex]

This equation's solution using the initial condition

T(0) = T0

T(t) = Tr + (T0 - Tr)

At the moment, the cooling rate

k × (T(t) - Tr)

We can write that according to the text

T1 = Tr + (T0 - Tr)

Solving the system of equations generates the unknown time:

t = [tex]\frac{69-20}{1} (ln\frac{95-20}{69-20})[/tex]

t = 21 min.

Hence the coffee will reach the temperature 67°C in 21 minutes.

Learn more about Thermodynamics at

https://brainly.com/question/1604031?referrer=searchResults

#SPJ1

Write an equation for the line graphed below.

Answers

Answer:

y=-4/3x

Step-by-step explanation:

There is no b because it starts at the origin.

Answer:

-4/3x is the correct answer as provided by IceJadeKitsune

I am merely providing an explanation in case you are curious

Step-by-step explanation:

The equation for a line in slope-intercept form is

y = mx + b

where m is the slope and b the y-intercept

The slope of a line, m can be determined by what is called the rise/run ratio

The process is as follows

Take any two convenient points on the line and note their coordinatesLet's label the points as (x1, y1) and (x2, y2)The rise is the difference in the y values = y2 - y1 (The run is the difference in the x values = x2 - x1Divide this rise over run and you get the slope, mTo get b, find where the line crossed the y-axis and the value of b will be the value of y at that point.

The value of x will be 0 at the y-intercept

From the graph choose points origin (0, 0)  which is one point where the line crosses the y axis

From the graph we see that another distinct point where the line passes is at x = -3, y = 4 or the point(-3, 4)

Having got these two points we calculate
rise = 4 - 0 = 4
run = -3 - 0 = -3
Slope m = -4/3

Slope is negative because as y increases, x decreases

So the equation is y = -4/3x + b

Looking at the graph, we see that b = 0 since the graph passes through the origin as correctly stated by  IceJadeKitsune


Therefore the equation of the line is [tex]\boxed{\bold{y = -\dfrac{4}{3}x}}[/tex]

find the height of a square pyramid with V of 60 cm3 and base side length S of 6cm

Answers

Using Volume formula, the height of the square pyramid is 5 cm.

What is volume of a square pyramid?

If the side of the square base of the pyramid is b and height of the pyramid be h

Volume of the square pyramid , V is given by the below formula.

V = 1/3(b²h)

Given

Volume of the square pyramid, V = 60 cm³

base side is given , b = 6 cm

let Height of the square pyramid be h

⇒ V =  1/3(b²h)

⇒ 3V = b²h

⇒3V/b² = h

⇒ 3(60)/ 6² = h

⇒ h = 180/36 = 5

Therefore, the height of the given square pyramid id 5 cm.

Also, learn more about Volume of Pyramid from the link below:

https://brainly.com/question/17615619

#SPJ1

A poster is 3 feet wide and 12 feet long. What are the dimensions if the poster is enlarged by a factor of 5/2?

Answers

To be able to get the enlarged dimensions, we will multiply the dimension of the poster by the given scale factor 5/2.

We get,

[tex]\text{Width = 3 ft. x }\frac{5}{2}\text{ = }\frac{15}{2}\text{ ft. or 7.5 ft.}[/tex][tex]\text{Length = 12 ft. x }\frac{5}{2}\text{ = }\frac{60}{2}\text{ ft. or 30 ft.}[/tex]

Therefore, the enlarged dimension of the poster will be 7.5 ft. x 30 ft.

let f(x) =(4x"3+20)"2 and g(x) =4x"3+20.given that f(x)=(h°g)(x), find h(x)

Answers

[tex]h(x)=x^2[/tex]

Here, we want to find the function h(x)

From the question, we can see that the function f(x) is a composite function that was obtained by fitting g(x) into h(x)

What can we notice about g(x) and f(x)?

What we can see is that f(x) is the square of g(x)

Thus, what this mean is that h(x) = x^2

Enter the value of b when the expression 1/2x + b is equivalent to 1/4(2x+3)

Answers

Answer:

b = 3/4

Step-by-step explanation:

[tex]\frac{1}{2}x + b=\frac{1}{4} (2x+3)[/tex]

Distribute.

[tex]\frac{1}{2}x + b=\frac{1}{2} x+\frac{3}{4}[/tex]

Subtract 1/2x from both sides to isolate the b.

[tex]b=\frac{1}{2} x+\frac{3}{4}-\frac{1}{2}x[/tex]

1/2x and -1/2x cancel each other out.

[tex]b=\frac{3}{4}[/tex]

So b = 3/4

I believe you start with 2.4 million has initial start value? Not sure cause it worded weirdly and English not very good

Answers

2,466,000

Here, we want to know what is to be used as the initial value

The general exponential equation should be in the form;

[tex]P=I(1+R)^n[/tex]

where P is the population at a certain year

I is the initial population which is 2,466,000 in this case

R is the rate of increase

n is the number of years

What does the dashers part of the figure represent

Answers

The figure represents : Line

What is a Line?

A line is an object in geometry that is indefinitely long and has neither breadth nor depth nor curvature. Since lines can exist in two, three, or higher dimensional environments, they are one-dimensional things. The term "line" can also be used to describe a line segment in daily life that contains two locations that serve as its ends.

from the figure we have to find that  whether its a line, line segment, vertex or ray

Line: A line is a perfectly straight, one-dimensional shape that extends infinitely in both directions and has no thickness. Sometimes a line is referred to as a straight line or, more formally, a right line.

Line segment: In other words, a line segment is a section or element of a line with two endpoints. A line segment, in contrast to a line, has a known length. A line segment's length can be calculated using either metric measurements like millimeters or centimeters or conventional measures like feet or inches.

Ray: A ray is a vector from a point to another point when seen as a vector. A ray is typically viewed in geometry as a half-infinite line, or half-line, with one of the two points and assumed to be at infinity.

Vertex: A vertex is a point on a polygon where two rays or line segments meet, the sides, or the edges of the object come together. Vertex is the plural form of vertices.

The figure represents a Line

Hence,  The dashers part of the figure represent a Line

To learn more about Line click on the link

https://brainly.com/question/2437195

#SPJ9

Other Questions
what is ST? I've included a photo of the problem The synthesis of sodium chloride according to the reaction: ___ Na + ___ Cl2 ___ NaCl.a. What volume of chlorine gas at STP is necessary for the complete reaction of 4.85 grams of sodium metal. 9x+7 what it is bc (9x+7)(-3x+20) is 39? 4. A bicyclist bikes to the West at a constant speed for 30 minutes before turning South and continuing at the same speed for an hour. Is the bicyclist accelerating? Why or why not? solve for x x/3 = 10 Immediately, theenchanting womanbegan doting on us, likea tiny yellow butterfly,fluttering about.Figurative Language 10. Food producers do not needmake sugar.a) carbon dioxideb) nitrogen dioxidec) waterd) lightto a grocery store wants to examine the relationship between the sales amounts each day at two different locations, store a and store b. the sales amount each day, in dollars, was recorded for 10 days at each store. the least-squares regression line is y Sue traveled 96 miles in 12 hours. A unit rate to describe Sue's travel is 8 miles per hour. What is another unit rate in this situation? True or FalseThe Georgia Platform required the enforcement of the Fugitive Slave Act, an agreement to abide bythe Compromise of 1850, and stopped Georgia from seceding from the Union.O TrueO False my opponent will say that recycling is in an issue here but that's just because they don't care about the sea turtles or other marine life affected by plastic in the ocean what is a logical fallacy how many books does matt have to sell to make 30$ for the week negative impact of cellular base stations on public health what is the most appropriate fluid intake strategy be for a team sport athlete who expects to lose 2.5 liters of sweat (about 3% of body mass) during a practice? What what solid is generated when the semi circle is rotated about the line a restaurant sells pizza for $12.76/slice. expenses for the restaurant include raw material for pizza at $7.76 per slice, $116.00 as monthly rental and $54.00 as monthly insurance. how many slices should the restaurant sell in a month to break even? don't round to whole slices. 2|x+3|+1>3what the soulution?? May I please get help with this. For I am confused and have tried multiple times Need help answering the following polynomial I need help with this practice problem The subject is trigonometry It asks to graph the function If you can use Desmos to graph